Give brainiest if right!


Marina spent $13.50 at the grocery store. She bought pears,

kiwis, and pineapples. Pears cost $0.50 each, pineapples cost

$1.50 each, and kiwis are $0.30 each. How many of each kind of

fruit did she buy if she bought 9 more pears than pineapples and

2 fewer kiwis than pears?

Answers

Answer 1

Answer:

Step-by-step explanation:

First we are going to set up our general equation (what is being asked of us). Marina bought pears, pineapples, and kiwis and spent $13.50 on them. The equation for that is

pi + pe + k = 13.5

Now we need to figure out how to eliminate most of those unknowns and put 2 of them in terms of the other 1. It looks like everything is based on the number of pineapples she bought. First it says she bought 9 more pears than pineapples, so obviously, there are more pears than pineapple, so

pe = pi + 9

And if she bought 2 fewer than kiwis thatn pears, and pears = pi + 9, then the number of kiwis she bought was

kiwis = (pi + 9) - 2 which simplifies down to pi + 7.

Now we'll put all of those into the equation:

pi + (pi + 9) + (pi + 7) = 13.5 What I have done is create an equation that is not parallel. In other words, I have the NUMBER of the kinds of fruit on one side of the equation, and the COST of the fruit on the other side, and that's not cool. We have to have EITHER a NUMBER of fruit equation OR a COST of fruit equation, but not both in the same equation. To amend that, we will figure in the cost of each of these kinds of fruit by the correcpsonding number of that kind of fruit. Pineapples cost $1.50 each, so the expression for pineapples is 1.5pi; pears cost $.50 each, so the expression for pears is.5(pi + 9); kiwis cost $.30 each, so the expression for kiwis is .3(pi + 7). NOW we can set up the equation:

1.5pi + .5(pi + 9) + .3(pi + 7) = 13.5 and simplify:

1.5pi + .5pi + 4.5 + .3pi + 2.1 = 13.5 and simplify some more by combining like terms:

2.3pi = 6.9 so

pi = 3. Ok we have 3 pineapples. Now we go back up to the expression for pears:

pe = pi + 9 so

pears = 12. Now we go back to the expression for kiwis:

k = pi + 7 so

kiwis = 10. And there you go!


Related Questions

The hypotenuse of a right triangle measures 14 cm and one of its legs measures 1 cm. Find the measure of the other leg. If necessary, round to the nearest tenth.

Answers

Answer:

b=14 cm

Step-by-step explanation:

Use pythagorean equation

A^2+b^2=c^2

1^2+b^2=14^2

1+b^2=196

b^2=195

b=13.964

Suppose the distributor charges the artist a $40.00 cost for distribution, and the streaming services pays $4.00 per unit. (Note: One unit = one thousand streams)

- - - - - - - - - - - - - - - - - - - - - - - - - - - - - - - - - - - - - - - - - - - - - - - - - - - - - - - - - - - -

Formula: y = 40x + 4 (Graph Attached)

- - - - - - - - - - - - - - - - - - - - - - - - - - - - - - - - - - - - - - - - - - - - - - - - - - - - - - - - - - - -

After how many streams will you pay for the distributor charges? (Hint: this is where the line crosses the x-axis, round to the nearest thousand)

Answers

Answer:

356 streams

Step-by-step explanation:

From the graph, you will see that the line cross the x-axis at x = 8.8

Substitute into the expression y = 40x + 4

y = 40(8.8)+4

y = 352 + 4

y = 356

Hence the distributor charges will be paid for after 356 streams

.........................................................

Answers

Answer:

..............................what this

what percent is equal to 7/25​

Answers

28% because 25x4=100 7x4=28

in a fruit punch drink,the 3 ingredients are apple juice,orange juice and cramberry juice.if 3/4 of the drink is apple juice and 1/10 is orange juice then write the ratio of cranberry juice to apple juice to orange juice in its simplest form​

Answers

Answer:

3 : 15 : 2

Step-by-step explanation:

Let cranberry juice = x,

3/4 + 1/10 + x = 1

x = 3/20

Ratio = cranberry : apple : orange

= 3/20 : 3/4 : 1/10

= 3 : 15 : 2 (Times everything with 20)

Which choice correctly shows the solution(s) of the equation x2 = 1442
A)
x= √144
B)
x=V12
X=-
-V144
D)
x = 1V144

Answers

Answer:

Step-by-step explanation:

If the 2s are exponents, you need to indicate this with "^":  

x^2 = 144^2 means x² = 144²

x = ±√144² = ±144

Answer:

Step-by-step explanation:

f the 2s are exponents, you need to indicate this with "^":  

x^2 = 144^2 means x² = 144²

x = ±√144² = ±144

According to government data, the probability than an adult never had the flu is 19%. You randomly select 70 adults and ask if he or she ever had the flu. Decide whether you can use the normal distribution to approximate the binomial distribution, If so, find the mean and standard deviation, If not, explain why. Round to the nearest hundredth when necessary.

Answers

Answer:

Since [tex]np \geq 10[/tex] and [tex]n(1-p) \geq 10[/tex], the normal distribution can be used to approximate the binomial distribution.

The mean is 13.3 and the standard deviation is 3.28.

Step-by-step explanation:

Binomial probability distribution

Probability of exactly x successes on n repeated trials, with p probability.

Can be approximated to a normal distribution, using the expected value and the standard deviation.

The expected value of the binomial distribution is:

[tex]E(X) = np[/tex]

The standard deviation of the binomial distribution is:

[tex]\sqrt{V(X)} = \sqrt{np(1-p)}[/tex]

Normal probability distribution

Problems of normally distributed distributions can be solved using the z-score formula.

In a set with mean [tex]\mu[/tex] and standard deviation [tex]\sigma[/tex], the z-score of a measure X is given by:

[tex]Z = \frac{X - \mu}{\sigma}[/tex]

The Z-score measures how many standard deviations the measure is from the mean. After finding the Z-score, we look at the z-score table and find the p-value associated with this z-score. This p-value is the probability that the value of the measure is smaller than X, that is, the percentile of X. Subtracting 1 by the p-value, we get the probability that the value of the measure is greater than X.

When we are approximating a binomial distribution to a normal one, we have that [tex]\mu = E(X)[/tex], [tex]\sigma = \sqrt{V(X)}[/tex], if [tex]np \geq 10[/tex] and [tex]n(1-p) \geq 10[/tex].

The probability than an adult never had the flu is 19%.

This means that [tex]p = 0.19[/tex]

You randomly select 70 adults and ask if he or she ever had the flu.

This means that [tex]n = 70[/tex]

Decide whether you can use the normal distribution to approximate the binomial distribution

[tex]np = 70*0.19 = 13.3 \geq 10[/tex]

[tex]n(1-p) = 70*0.81 = 56.7 \geq 10[/tex]

Since [tex]np \geq 10[/tex] and [tex]n(1-p) \geq 10[/tex], the normal distribution can be used to approximate the binomial distribution.

Mean:

[tex]\mu = E(X) = np = 70*0.19 = 13.3[/tex]

Standard deviation:

[tex]\sigma = \sqrt{V(X)} = \sqrt{np(1-p)} = \sqrt{70*0.19*0.81} = 3.28[/tex]

The mean is 13.3 and the standard deviation is 3.28.

integration of 3^x (1-3^(x+1)^9)dx​

Answers

Step-by-step explanation:

the answer is in picture

x(x+3)(x+3)=0

solve the equation only one answer

Answers

Answer:

0

Step-by-step explanation:

it says the answer is zero


3,
If an angle measures 29°, find its supplement.
7
4
Kelsey is drawing a triangle with angle measures of 128° and 10°. What is the measure of
the missing angle?
A
1280
10°
В
not to scale
7.6.2 DOK

Answers

9514 1404 393

Answer:

  3. 151°

  4. 42°

Step-by-step explanation:

3. The measure of the supplement is found by subtracting the angle from 180°.

  supplement of 29° = 180° -29° = 151°

__

4. The total of angles in a triangle is 180°, so the third one can be found by subtracting the other two from 180°.

  third angle = 180° -128° -10° = 42°

Please help im begging you

Find the domain of the function expressed by the formula:
y = 1/x - 7

Answers

Answer:

the domain is ALL reals numbers except ZERO

- ∞ < x < 0    ∪   0 <  x < ∞

Step-by-step explanation:

Answer:

(-∞,0) ∪ (0,∞), {x|x≠0}

Step-by-step explanation:

I think this is it. Im not completely sure though

Find the lengths of AD, EF, and BC in the trapezoid below.

Answers

Answer:

Step-by-step explanation:

Segment EF is mid-segment of ABCD ⇒ ( 2x - 4 ) + ( x - 5 ) = 2x

x - 9 = 0

x = 9

AD = 4

EF = 9

BC = 14

The length of segments AD, EF, and BC in the trapezoid are 4, 9 and 14 respectively

What is Coordinate Geometry?

A coordinate geometry is a branch of geometry where the position of the points on the plane is defined with the help of an ordered pair of numbers also known as coordinates.

We have to find the lengths of AD, EF, and BC in the trapezoid

Segment EF is mid-segment of ABCD

So ( 2x - 4 ) + ( x - 5 ) = 2x

Now let us solve for x

2x-4+x-5=2x

Combine the like terms

x-9=0

x=9

So AD =x-5

=9-5= 4

EF = 9

BC = 2x-4

=18-4

=14

Hence, the length of segments AD, EF, and BC in the trapezoid are 4, 9 and 14 respectively

To learn more on Coordinate Geometry click:

brainly.com/question/27326241

#SPJ2

What is the shape of the cross section?

Answers

Answer:

Step-by-step explanation:

Triangular cross-section.

Answer:

it is a triangle cross-section

hope this answer helps you

Plz make me a brainlist

If f(x) = 4x + 5 and fog(x) = 8x + 13 then find g(x).​

Answers

Answer:

given

f(x).4x+5

fog(x).8x+13

now

fog(x):8x+13

4x+5(g(x)):: 8x+13

g(x):: 8x+13/4x+5

Answer:

g(x) = 2x + 2

Step-by-step explanation:

One is given the following information:

f(x) = 4x + 5f o g (x) = 8x + 13

One is asked to find the following:

g(x)

Remember, (f o g (x)) is another way of representing a composite function. A more visual way of representing this composite function is the following (f(g(x)). In essence, one substitutes the function (g(x)) into the function (f(x)) in places of the varaible (x). Thus, represent this in the form of an equation:

f(g(x)) = 8x + 13

Substitute the given infromation into the equation:

4(g(x)) + 5 = 8x + 13

Solve for (g(x)) in terms of (x). Remember to treat (g(x)) as a single parameter:

4(g(x)) + 5 = 8x + 13

Inverse operations,

4(g(x)) + 5 = 8x + 13

4(g(x)) = 8x + 8

g(x) = (8x + 8) ÷ 4

g(x) = 2x + 2

From the observation deck of a skyscraper, Isabella measures a 67
angle of depression to a ship in the harbor below. If the observation deck is 824 feet high, what is the horizontal distance from the base of the skyscraper out to the ship? Round your answer to the nearest tenth of a foot if necessary.

Answers

The horizontal distance from the base of the skyscraper out to the ship is 349.8feet

Angle of elevation and depression

The angle situated above the hill is known as  the angle of depression

Given the following parameters

Height of the harbor = 824 feet

Angle of depression = 67degrees

According to SOH CAH TOA identity:

tan 67 = opp/adj
tan 67 = 824/d

d = 824/tan67
d = 349.8 feet

Hence the horizontal distance from the base of the skyscraper out to the ship is 349.8feet

Learn more on angle of depression here: https://brainly.com/question/12483071

formula of a square minus b square​

Answers

Answer:

(a+b)(a-b)

Step-by-step explanation:

[tex]\\ \sf\longmapsto (a+b)(a-b)[/tex]

[tex]\\ \sf\longmapsto a(a-b)+b(a-b)[/tex]

[tex]\\ \sf\longmapsto a^2-ab+ba-b^2[/tex]

[tex]\\ \sf\longmapsto a^2-ab+ab-b^2[/tex]

[tex]\\ \sf\longmapsto a^2-b^2[/tex]

According to the question

[tex]\large\bf{\orange{ \implies}} \: \tt \: {a}^{2} \: - \: {b}^{2} [/tex]

[tex]\large\bf{\pink{ \implies}} \: \tt \: (a + b) \quad \: (a - b)[/tex]

[tex]\large\bf{\pink{ \implies}} \: \tt \: a \: (a - b) \quad \: b \: (a - b)[/tex]

[tex]\large\bf{\pink{ \implies}} \: \tt \: {a}^{2} \: - \: ab \: + \: ba \: - \: {b}^{2} [/tex]

[tex]\large\bf{\pink{ \implies}} \: \tt \: {a}^{2} \: - \: ab \: + \: ab \: - \: {b}^{2} [/tex]

[tex]\large\bf{\pink{ \implies}} \: \tt \: {a}^{2} \: - \: \cancel{ab} \: + \: \cancel{ab} \: - \: {b}^{2} [/tex]

[tex]\large\bf{\pink{ \implies}} \: \tt \: {a}^{2} \: - \: {b}^{2} [/tex]


Suppose $12,000 is deposited into an account paying 5.5% interest, compounded continuously.
How much money is in the account after five years if no withdrawals or additional deposits are
made?

Answers

Answer:

$15798.4

Step-by-step explanation:

We will have to use this formula A = Peᵃᵇ

A = Final amount

P = Initial amount (12,000)

e = Mathematical constant: 2.7183

a = Interest rate (5.5% or 0.055)

b = Years

So our equation will look like this

A = 12,000e⁵ ⁰·⁵⁵

A = 12,000(2.7183)·²⁷⁵

A = 12,000(1.316533)

A = 15798.396

12. Convert 30.283° into a degree-minute-second format.
O A. 18° 16' 98"
B. 18° 28' 30"
C. 30° 16' 58"
D. 30° 28' 30"

Answers

D 30°28’30”
Non of the above it’s correct.
The correct answer is 283/60 =4°71’
71-60=1

4°+30°+1°= 35°

Answer 35°01’00”

The angle of 30.283° in a degree-minute-second format will be 30° 16' 58". Then the correct option is C.

What is conversion?

Unit modification is the process of converting the measurement of a given amount between various units, often by multiplicative constants that alter the value of the calculated quantity without altering its impacts.

The inclination is the separation seen between planes or vectors that meet. Degrees are another way to indicate the slope. For a full rotation, the angle is 360°.

The angle is given below.

⇒ 30.283°

Convert 30.283° into a degree-minute-second format. Then we have

⇒ 30° (0.83 x 60')

⇒ 30° 16.98'

⇒ 30° 16' (0.98 x 60'')

⇒ 30° 16' 58"

The angle of 30.283° in a degree-minute-second format will be 30° 16' 58". Then the correct option is C.

More about the conversion link is given below.

https://brainly.com/question/9414705

#SPJ2

Rewrite the polynomial in the form ax+by+c and then identify the values of a, b, and c.

x -- 1 -- 2y

Answers

Answer:

a=1, b=-2,c=-1

Step-by-step explanation:

1*(x)+(-2)*y+(-1)*1. a=1, b=-2,c=-1

Mary is thinking of a mystery number. She reduces it by 15% then subtracts 5. The result is 29. Determine the mystery number

Answers

Answer:

40

Step-by-step explanation:

Let x represent the mystery number.

Create an equation to represent the situation, then solve for x:

0.85x - 5 = 29

0.85x = 34

x = 40

So, the mystery number is 40.

- 2/3 (2 - 1/5) use distributive property

Answers

Answer:

-6/5

Step-by-step explanation:

- 2/3 (2 - 1/5)

Distribute

-2/3 *2 -2/3 *(-1/5)

-4/3 + 2/15

Get a common denominator

-4/3 *5/5 +2/15

-20/15 +2/15

-18/15

Simplify

-6/5

I need help completing this problem ASAP

Answers

4/(√x - √(x - 2)) × (√x + √(x - 2))/(√x + √(x - 2))

= 4 (√x + √(x - 2)) / ((√x)² - (√(x - 2))²)

= 4 (√x + √(x - 2)) / (x - (x - 2))

= 4 (√x + √(x - 2)) / (x - x + 2)

= 4 (√x + √(x - 2)) / 2

= 2 (√x + √(x - 2))

Carol is having a hard time understanding the central limit theorem, so she decides to do her own experiment using the class data survey collected at the beginning of class on the number of hours a student takes during her Spring 2019 BUSI 2305 course. The data file has a total number of 54 students where the average is 10.8 with a standard deviation of 3.15. She sets out to collect the mean on 8 samples of 6 students. Based on this what are the total possible samples that could occur based on the population

Answers

Answer:

25827165

Step-by-step explanation:

from the question that we have here

the total population = 54 students

the sample size = 6 students

So given this information carol has to pick the total samples from the 54 students that we have here

the total ways that she has to do this

= 54 combination 6

= 54C6

= [tex]\frac{54!}{(54-6)!6!}[/tex]

= 25827165

this is the total number of possible samples that could occur given the total population of 54 students.

HELP NEEDED PLEASE!!!!!

Answers

Answer:

1^1 + 0^1 =1

Step-by-step explanation:

sin^2 theta + cos^2 theta = 1

sin^2 (pi/2)   + cos^2  (pi/2)  =1

1^1 + 0^1 =1

What is the common ratio for this geometric sequence?
27, 9, 3, 1, ...

Answers

Answer:

1/3

Step-by-step explanation:

common ratio is

9÷27=1/3

3÷9=1/3

1÷3=1/3

therefore common ratio is 1/3

Answer: 1/3

Step-by-step explanation:

Let us confirm that this is a geometric sequence. 9/27 = 1/3 and 3/9 = 1/3. Thus, the common ratio is 1/3.

The equation ^2 −4+^2 +2=−4
a. Is a parabola
b. Is an ellipse
c. Is a hyperbola
d. Is a circle
e. None of the above

Answers

Answer:

Step-by-step explanation:

None. Your notation is unclear.

i provided the question

Answers

Answer:

(0, 3)

Step-by-step explanation:

y = 3 is the horizontal tangent to y = x^2+3, and passes the parobala at (0, 3)

Select the correct answer.
Given the following formula, solve for l.



A.

B.

C.

D.

Answers

Please rewrite the question or add a picture for it

Answer:

c

Step-by-step explanation:

took the test so i assume its this question

If (x+2) is a Factor x^3 + 2x^2 + 2x + k then find the value of K.​

Answers

Answer:

4

Step-by-step explanation:

if x+2 is a factor of the above expression then,

x=-2

so putting the value of x in above expression we get,

(-2)^3+2×(-2)^2+2×(-2)+k=0

or,-8+8-4+k=0

k=4

Answer:

Step-by-step explanation:

If (x + 2) is a factor of a polynomial then ( - 2 ) is the zero of that polynomial ⇒ ( - 2 )³ + 2( - 2 )² + 2( - 2 ) + k = 0 ⇒ k = - 4

Find the function G defined by G(x) =5x+3 find G(-1)

Answers

Answer:

G(-1) = -2

General Formulas and Concepts:

Pre-Algebra

Order of Operations: BPEMDAS

Brackets Parenthesis Exponents Multiplication Division Addition Subtraction Left to Right

Algebra I

Functions

Function Notation

Step-by-step explanation:

Step 1: Define

Identify

G(x) = 5x + 3

Step 2: Evaluate

Substitute in x [Function G(x)]:                                                                         G(-1) = 5(-1) + 3Multiply:                                                                                                             G(-1) = -5 + 3Add:                                                                                                                   G(-1) = -2

Answer:

G = -2

Step-by-step explanation:

Plug in -1 for x.

5(-1) + 3

-5 + 3

-2

G = -2

Other Questions
please help me solve the equations, I re-wrote the question under each of them so you can read it better. Ty Solve - 2x - 7 > X+ 8.O A. x-3 [tex]\lim_{x \to \4} x^{2} -3x[/tex] Instructions: Find the missing side. Round your answer to the nearesttenth.4966X The robotics team purchased 3 androids for the purpose of programming. Each of the robots was $398, which included tax. If the tax rate is 8%, what is the TOTAL TAX to be paid? Need helppppp Im stuckkkk how many terms are there in the series. 201.208.215......319 I need help pls help me in geometry homework Deanna, the human resource manager of a firm, is planning how to support a strategy of empowering employees. Currently, jobs are grouped based on different functions, such as customer service, manufacturing, and packaging. Deanna is considering a redesign in which members of various functions work in divisions responsible for a particular customer group. In what way could this redesign support the goals for employee empowerment How much time do you spend .....TV every day ? A watch B to watch C watching D in watching 1.Explain how is small intestine designed to absorb digested food.2.Write two differences between the nutrition in amoeba and human beings.3.Fungus can be harmful and useful. Give an example showing both of these traits of fungus. mention the values that develop through olymic movement Optimism and courage help to tide over difficulties'. Substantiate the statement in the light of We're Not Afraid to Dieif We Can All Be Together What is Index Law 2?please give definition Create a program that uses a separate module to calculate sales tax and total after tax.Create a c++ program using console.h and console.cpp files that uses a separate module to calculate sales tax and total after tax.ConsoleSales Tax Calculator ENTER ITEMS (ENTER 0 TO END) Cost of item: 35.99 Cost of item: 27.50 Cost of item: 19.59 Cost of item: 0 Total: 83.08 Sales tax: 4.98 Total after tax: 88.06 Again? (y/n): y ENTER ITEMS (ENTER 0 TO END) Cost of item: 152.50 Cost of item: 59.80 Cost of item: 0 Total: 212.30 Sales tax: 12.74 Total after tax: 225.04 Again? (y/n): n Write as an equation: a triangle has two equal sides and a third side that is 15cm long. How long is each of the equal sides if the perimeter is 50? Type your answer into the box. X 30 Calculate the size of angle x. angle x = If there are 18 people in your class and you want to divide the class into programming teams of 3members, you can compute the number of different teams that can be arranged using this formula(n!/r!(nr)!). Someone plz help me on this one The philosophy of John Locke strongly supported the idea that: Group of answer choices political leaders are born, not made. property rights should be subordinated to human rights. reason is an inadequate guide in establishing a political order. equality of goods and income is necessary for the political order. government ought to rest on the voluntary consent of the governed.